2
$\begingroup$

It seems like the sum $S(n)$ should be possible to upperbound by an expression of the form ${\mathcal O}(n^a\cdot \log^b(n))$ as $n \rightarrow \infty$: $$ S(n)\stackrel{\triangle}{=}\sum_{1\leq x \neq y\leq n} \frac{gcd(x,y)^2}{x y}. $$ Any pointers, ideas, appreciated. Since $gcd(x,y) \leq \min(x,y)$ an upper bound is given by $$ S(n) \leq \sum_{1\leq x \neq y\leq n} \frac{\min(x,y)^2}{x y} = 2 \sum_{1\leq x < y\leq n} \frac{x}{y} $$ which can be rewritten as $$ S(n) \leq 2 \sum_{1\leq x \leq n-1} x \sum_{x+1 \leq y \leq n} \frac{1}{y} = 2 \sum_{1\leq x \leq n-1} x (H(n)-H(x)), $$ where $H(n)$ denotes the harmonic sum. This then gives $$ S(n) \leq 2 H(n) \left[\sum_{1\leq x \leq n-1} x\right] -2 \left[\sum_{1\leq x \leq n-1} x H(x)\right] $$ or $$ S(n) \leq n(n-1) H(n) - 2 (1\times H(1)+2 \times H(2)+\cdots+(n-1) \times H(n-1)) $$ or to a first order approximation $$ S(n) \leq n(n-1) H(n) - 2 \sum_{1\leq x \leq n-1} x \log x \approx n^2 \log n - 2 (1+\sum_{2\leq x \leq n-1} x \log x) . $$ An application of the arithmetic geometric mean inequality an Stirling seems to give an estimate of $(n-1)^2$ for the subtracted sum while an integral approximation says that the subtracted sum is ${\cal O}(n^2 \log n)$ which would cancel the first term! Using the estimate $\log n + \gamma$ for the Harmonic sum then leaves us with $S(n) \leq \gamma n(n-1).$

Is there a more accurate estimate?

$\endgroup$
1
  • $\begingroup$ More easily, fixing y and summing 2x/y for x from 1 to y-1 gives y-1, leading to n choose 2 as an upper bound on S(n). Lucia's approach of summing on lines (at,bt) does lead to a better estimate, both on upper and lower bounds. $\endgroup$ Feb 15, 2014 at 18:42

1 Answer 1

8
$\begingroup$

Suppose that $x <y$. Write $x=ga$ and $y=gb$ where $g$ is the gcd of $x$ and $y$; thus $a$ and $b$ are coprime with $a<b$, and $g\le n/b$. So $$ S(n) = 2 \sum_{\substack{{a<b \le n} \\ {(a,b)=1}}} \frac{1}{ab} \sum_{g\le n/b} 1 \le 2\sum_{\substack{{a<b\le n} \\ {(a,b)=1}}} \frac{1}{ab} \frac{n}{b}. $$ Now ignore the condition that $(a,b)=1$, getting (using $\sum_{b>a} 1/b^2 \le 1/a$) $$ S(n) \le 2n \sum_{a\le n} \frac{1}{a} \sum_{a<b\le n} \frac{1}{b^2} \le 2n\sum_{a\le n} \frac{1}{a^2} \le \frac{\pi^2}{3} n. $$ With more effort you can prove that $S(n) \sim Cn$ for some constant $C$.

Edit: The OP Kodlu noted that numerically the constant $C$ seemed to be $2$, which was surprising to me. So here's a sketch as to why the constant comes out to be $2$. The argument above is pretty accurate, and is essentially an asymptotic. Thus from above one finds that $$ C = 2\sum_{\substack{{1\le a<b } \\ {(a,b)=1} }} \frac{1}{ab^2}. $$ Now use Mobius inversion $\sum_{d|(a,b)} \mu(d) =1$ if $(a,b)=1$ and zero otherwise. Thus, with $a=dr$ and $b=ds$,
$$ C= 2\sum_{1\le a<b} \frac{1}{ab^2} \sum_{d|(a,b)} \mu(d) =2 \sum_{d=1}^{\infty} \frac{\mu(d)}{d^3} \sum_{1\le r<s} \frac{1}{rs^2}. $$ The sum over $d$ is $1/\zeta(3)$, and the sum over $r$ and $s$ is the multizeta value $\zeta(2,1)$ which Euler showed equals $\zeta(3)$. Thus $C=2$ and $S(n) \sim 2n$ (and in fact the argument above gives this as an upper bound always, as seen in the numerics).

$\endgroup$
3
  • 1
    $\begingroup$ Thanks, that's very helpful. I will do some numerical experiments as well, but is the constant C likely to be below 1? It seems somewhat unlikely $\endgroup$
    – kodlu
    Feb 17, 2014 at 0:33
  • $\begingroup$ I spoke too soon, constant seems to approach 2 from below but not quite reach it for n up to 5000 (granted I didn't check all values beyond about 500) but only checked at regular intervals. Some values of F(n)=S(n)/n are F(1000)=1.97248, F(2000)=1.98395, F(5000)=1.99266, F(10000)=1.99585, according to Mathematica. I wonder if bounds on C are known? $\endgroup$
    – kodlu
    Feb 17, 2014 at 0:59
  • 1
    $\begingroup$ @kodlu: That's very interesting and surprising to me. But I see now that the constant is indeed $2$ by Euler's formula that the multizeta value $\zeta(2,1)$ equals $\zeta(3)$. I'll edit my answer to include this. $\endgroup$
    – Lucia
    Feb 17, 2014 at 1:04

Your Answer

By clicking “Post Your Answer”, you agree to our terms of service and acknowledge you have read our privacy policy.

Not the answer you're looking for? Browse other questions tagged or ask your own question.